Linear algebra - dimension and intersection

Click For Summary
The discussion centers on proving that the codimension of the intersection of multiple subspaces in a finite-dimensional vector space is less than or equal to the sum of their codimensions. The key formula used is that the codimension of a subspace W in V is defined as codim(W) = dim(V) - dim(W). The participant is exploring the proof by induction and considering specific cases, such as when the dimensions of the subspaces are equal or when they intersect at the zero vector. They also analyze examples in R3, illustrating how the dimensions and codimensions relate in practical scenarios. The conversation highlights the complexity of the proof and the need for careful algebraic manipulation to establish the result.
zwingtip
Messages
20
Reaction score
0

Homework Statement


let V be a finite dimensional vector space of dimension n. For W \leq V define the codimension of W in V to be codim(W) = dim(V) - dim(W). Let W_i, 1 \leq i \leq r be subspaces of V and S = \cap_{i=1}^{r}W_i. Prove:

codim(S) \leq \sum_{i=1}^{r} codim(W_i)

Homework Equations


dim(U + V) = dim(U) + dim(V) - dim(U \cap V)<br /> \sum_{i=1}^{r} codim(W_i) = \sum_{i=1}^{r} (n - dim(W_i)

The Attempt at a Solution



I'm completely lost here. I know I need to prove this by induction. Any tips to point me in the right direction? Can I use the fact that codim(S) = n - dim(\cap_{i=1}^{r} W_i and dim(U \cap V) &lt; dim(U) + dim(V) assuming dim(U + V) \neq 0?
 
Physics news on Phys.org
only had a quick look, but not convinced this is true..

take the case when say dim(W1) = dim(W2) = 1, in Rn, then codim(W1) = codim(W2) = n-1

now say the intersection S is only the zero vector then dim(S) = 0 and codim(S) = n >n-1> Can up with oher higherdimensional arguments similarly
 
Still doesn't make sense to me. What if the intersection contains more than just the zero vector? Sorry for being stupid. Help?
 
actually i think my exampel was wrong as
codim(S) = n < codim(W1) + codim(W1) = 2n-2, which works for for n>=3

now take R3, say you have 2 planes P1 & P2 that intersect in a line L, then
dim(R3) = 3
dim(P1) = dim(P2) = 3, then codim(P1) = codim(P2) = 2
dim(L) = 1, codim(L) = 2

so
codim(L) = 2 <= codim(P1) + codim(P2) = 1 + 1 = 2
 
I think I've got it with an induction proof from the base cases r=1, r=2 and the inductive step being that it holds for r>2 then manipulating it algebraically for an entire page of 5mm graph paper. THanks.
 
Question: A clock's minute hand has length 4 and its hour hand has length 3. What is the distance between the tips at the moment when it is increasing most rapidly?(Putnam Exam Question) Answer: Making assumption that both the hands moves at constant angular velocities, the answer is ## \sqrt{7} .## But don't you think this assumption is somewhat doubtful and wrong?

Similar threads

Replies
1
Views
1K
Replies
15
Views
2K
Replies
34
Views
3K
  • · Replies 10 ·
Replies
10
Views
3K
Replies
8
Views
2K
  • · Replies 2 ·
Replies
2
Views
3K
Replies
1
Views
2K
  • · Replies 1 ·
Replies
1
Views
1K
  • · Replies 4 ·
Replies
4
Views
2K
  • · Replies 5 ·
Replies
5
Views
2K